2001 AIME I Problems/Problem 2

Revision as of 03:31, 15 March 2008 by I like pie (talk | contribs)

Problem

A finite set $\mathcal{S}$ of distinct real numbers has the following properties: the mean of $\mathcal{S}\cup\{1\}$ is $13$ less than the mean of $\mathcal{S}$, and the mean of $\mathcal{S}\cup\{2001\}$ is $27$ more than the mean of $\mathcal{S}$. Find the mean of $\mathcal{S}$.

Solution

Let $x$ be the mean of $\mathcal{S}$. Let $a$ be the number of elements in $\mathcal{S}$. Then, \[\frac{ax+1}{a+1}=x-13\] and \[\frac{ax+2001}{a+1}=x+27\] \[\frac{ax+2001}{a+1}-40=\frac{ax+1}{a+1}\] \[\frac{2000}{a+1}=40\] so \[2000=40(a+1)\] \[a=49\] We plug that into our very first formula, and get: \[\frac{49x+1}{50}=x-13\] \[49x+1=50x-650\] \[x=651\]

See Also

2001 AIME I (ProblemsAnswer KeyResources)
Preceded by
Problem 1
Followed by
Problem 3
1 2 3 4 5 6 7 8 9 10 11 12 13 14 15
All AIME Problems and Solutions